« first day (893 days earlier)      last day (4019 days later) » 

12:28 AM
0
Q: Kramers-Kronig Mathematica code

o_0I'm trying to write Mathematica code to use Kramers-Kronig on a csv file with 2 columns. Column 1 is h(eV). It goes from 0 to 6, in increments of 0.1. Column 2 is alpha (cm^-1). For some reason, my code just outputs a blank graph. Can someone please tell me what's wrong with it? Here's the equa...

Off topic I think? VTC or flag if you agree!
 
12:55 AM
1
Q: Who is the target audience for these answers?

NickI was looking through questions on the home page, and I noticed this: Does entanglement not immediately contradict the theory of special relativity? Now, it's obvious from the way the poster posed his original question that the top answer is going to make almost no sense to him whatsoever; it's...

 
1:42 AM
@Manishearth Ahh... Okay ;-)
@DavidZaslavsky Like I do everytime ;-)
 
2:17 AM
What I do now to get this question seen?physics.stackexchange.com/questions/60974/… I've alredy posted it second time after deleting first.
 
2:27 AM
Hi all! First time using this room. I have a question that's poorly developed; I'm hoping you can help me flesh it out or pre-empt it completely.
I've recently found out something seemingly major that I don't understand regarding optics.
Everything I (thought) I understood about magnification can be described with the concepts of refractive index (to explain the how light rays get deflected by lenses) and the relative distances of object, lens, and viewing plane. I guess that's called Gaussian optics. Ray tracing and the like.
When working with a transmission electron microscope I learned that there's a relationship between the image plane and the plane located at the objective aperture that's confusing me. I think there's something basic I'm missing. Apparently, if one moved the image plane to the objective aperture's plane (close to where the objective [focusing] lens is) one would see a Fourier transform of the image.
So my ill-formed question is, do magnification lenses perform optical transforms on the object they're imaging? I always thought of diffraction and magnification as being two different scenarios. but maybe that's because I'm thinking in terms of rays and refractive indexes in one case, and scattering events in the other case. I'm guessing that both phenomena can be described in terms of scattering?
 
@Ryan I don't think I can help you. If you make a diagram of your understanding and your question for a question post you're likely to get a good response though.
 
2:43 AM
@Brandon thanks. I guess I should clarify that optical transform is short of Fourier transform effected by optics. Like if you take an FT of a image.
Maybe it's related to this post: physics.stackexchange.com/questions/23286/…
 
3:35 AM
@Ryan Perhaps so. Take a look at that post and see if it helps you at all. If you're still confused after that, I'd say you're pretty close to having a good question for the main site.
Just polish up what you've written here and it should be fine.
@exploringnet well, you've already answered your own question, so I'm not sure what else needs to be done about it
And now that I look at it, that's one of those no-effort homework questions we don't allow here, so I'm closing it.
 
user54412
3:49 AM
@Ryan That post you linked has possibly the most confusing exposition of optics I've ever seen - it's no wonder that doesn't answer any particular question of yours
 
user54412
so yeah, go ahead and post your question - i'm curious to see what answers it gets
 
5:05 AM
Thanks everyone. I'll make some diagrams and open a question.
 
 
8 hours later…
12:38 PM
3
A: Determine KE of electron given momentum & mass

UnkleRhaukus$p=2.38 \times 10^{-24}\left[\frac{\text {kg m}}{\text s}\right]$ $m_e=9.11\times10^{-31} [\text {kg}]$ $$\begin{align*} KE = \frac{p^2}{2m} &=\frac{\left(2.38\times10^{-24}\left[\frac{\text{kg m}}{\text s}\right]\right)^2}{2\times9.11\times10^{-31}[\text{kg}]}\\&=\frac{2.38^2\times10^{-24\time...

Anyone to say whether this answer is okay..???
While I truly encourage this answer, I'm quite suspicious whether it helps solving it..? :O — Crazy Buddy 51 secs ago
 
12:59 PM
BTW, Doesn't this answer plagiarize Nic's answer..? (though not completely)??
 
Hope everyone see both the posts..!! ;-)
 
1:18 PM
And this too...
This from this..!
I'm quite worried that it's the same user..! :(
I have a suspicion on the second post because, the other one has got a vote (which makes me suspicious that it was answered at first)
Twin posts by different authors..? Is that OK at different times..?
 
2:02 PM
Okay... I get a lot of questions today..! :P
At first, I don't respond much to "thank you" comments.
But nowadays, I've seen many responding to these comments as say - "you're welcome"...
I don't know why my brain got practiced with the environment, but now - I'm inserting comments like "you're welcome"...
While this is a nice habit, it also makes me suspicious that whether this dumps below the post..?
Is it OK to say "you're welcome" to "thank you" comments..? :P
 
 
5 hours later…
6:40 PM
0
Q: Should we have a badge / recognition of well-cited answers? Questions?

NewAlexandriaIt has been said that "citations are the source code of Physics.SE" References are how citations are added to a posting. Should we recognize good citation/reference practices via: a badge after a fixed number of answers, voted to N-score, each with at least X references a badge when total cit...

 
@CrazyBuddy yes
 
 
2 hours later…
8:36 PM
5
Q: Why is a black hole black?

user8784According to Newtonian mechanics, $v^2=\frac {G.M}{r}$ and according to special theory of relativity nothing could move faster than the speed of light, $$v^2 \le c^2$$ That means: $$\frac {G.M}{r} \le c^2$$ Then, how can we say that: a black hole is a region of spacetime where gravity is so ...

I just approved that edit but I'm having second thoughts about whether it really helps the question. Someone else might want to take a look at that.
 
I doubt I can contribute anything but I'll look.
the edit actually changes the meaning of the question and is wrong
Looking at the edit history, I can't figure out what the intention of the original question was. A couple edits have changed the meaning of the question. I think it should be rolled back to edit 8.
 
@DavidZaslavsky @BrandonEnright OP's original question is hopelessly backwards. Luckily OP is gone, so we are free to make a better question (the obvious one), which in turn, is what the answerers have already assumed and answered.
 
8:51 PM
Yeah the original question was a disaster.
We get some of the most poorly worded questions it seems.
 
@DavidZaslavsky @BrandonEnright I edited the question.
 
9:58 PM
does anybody know about the asynchronous motor
I need resources
any help
 
@pourjour are you talking about en.wikipedia.org/wiki/Induction_motor ?
 

« first day (893 days earlier)      last day (4019 days later) »